Đến nội dung

Ngockhanh99k48 nội dung

Có 114 mục bởi Ngockhanh99k48 (Tìm giới hạn từ 05-06-2020)



Sắp theo                Sắp xếp  

#696575 Tính định thức sau

Đã gửi bởi Ngockhanh99k48 on 14-11-2017 - 00:36 trong Đại số tuyến tính, Hình học giải tích

Tính định thức sau:

$M_n=\begin{pmatrix} a_1b_1 & a_1b_2 & a_1b_3 & ... & a_1b_n \\ a_1b_2 & a_2b_2 & a_2b_3 & ... & a_2b_n \\ a_1b_3 & a_2b_3 & a_3b_3 & ... & a_3b_n \\ ... & ... & ... & ... & ... \\ a_1b_n & a_2b_n & a_3b_n & ... & a_nb_n \end{pmatrix}$




#648205 $\frac{a}{\sqrt{a+b^{2}}...

Đã gửi bởi Ngockhanh99k48 on 06-08-2016 - 12:20 trong Bất đẳng thức và cực trị

Cái này mong bạn nhớ được đẳng thức:
$2(a^2+b^2+c^2)^2-6(ab^3+bc^3+ca^3) = \sum_{cyc} (a^2-c^2-ab-ac+2bc)^2$



#648203 CMR: ab^2 <=1/8

Đã gửi bởi Ngockhanh99k48 on 06-08-2016 - 12:08 trong Bất đẳng thức và cực trị

Bạn cứ khai triển tung hết ra, khi đó ta có hệ thức $b(2a+1)=1$. Do đó, sử dụng AM-GM ta có: $1=b^2(2a+1)^2 \geq 8ab^2$. Do đó ta có đpcm. Dấu bằng xảy ra khi $a=b=\frac{1}{2}$



#657911 Bài toán của thầy Trần Quang Hùng - THPT Chuyên KHTN

Đã gửi bởi Ngockhanh99k48 on 15-10-2016 - 14:40 trong Đại số

Chuyển về bài toán sau cho bạn tự chứng minh nhé :) : cho $\triangle ABC$ với đường cao $AD, BE, CF$. $EF$ cắt $BC$ tại $A_1$. Tương tự với $B_1, C_1$. Chứng minh trung điểm $AA_1, BB_1, CC_1$ thẳng hàng.



#658171 Tính: $\sum_{i=1}^{n}\frac{1}...

Đã gửi bởi Ngockhanh99k48 on 17-10-2016 - 09:51 trong Dãy số - Giới hạn

$\sum_{i=1}^{n} \frac{1}{\sqrt{u_{i}}+1} = \sum_{i=1}^{n} (\frac{1}{\sqrt{u_{i}}} - \frac{1}{\sqrt{u_{i+1}}}) = \frac{1}{\sqrt{u_{1}}} - \frac{1}{\sqrt{u_{n+1}}}$



#658176 Chứng minh rằng $\frac{1}{a^{2}+b^{2}+c^{2}}+\frac{1}{\sq...

Đã gửi bởi Ngockhanh99k48 on 17-10-2016 - 11:07 trong Bất đẳng thức - Cực trị

Sử dụng AM-GM ta có:
$\frac{1}{a^2+b^2+c^2} + \frac{1}{3\sqrt{abc}} +\frac{1}{3\sqrt{abc}} +\frac{1}{3\sqrt{abc}} \geq 4 \sqrt[4] {\frac{1}{a^2+b^2+c^2}.(\frac{1}{3\sqrt{abc}})^3} = 4 \sqrt[4]{\frac{1}{27(a^2+b^2+c^2)\sqrt{a^3b^3c^3}}}$
Cuối cùng ta chứng minh $27abc(a^2+b^2+c^2)\sqrt{abc} \leq 81$.
Theo AM-GM ta có $abc \leq 1$. Do đó
$27abc(a^2+b^2+c^2)(abc)^{\frac{1}{3}}.(abc)^{\frac{1}{6}}$ $\leq$ $9abc(a^2+b^2+c^2)(a+b+c)$ $\leq$ $3(a^2+b^2+c^2)(ab+bc+ca)^2$ $\leq$ $\frac{1}{9}[(a^2+b^2+c^2)+2(ab+bc+ca)]^3 = \frac{(a+b+c)^6}{9} =81$. Ta có đpcm. Dấu bằng xảy ra khi $a=b=c=1$



#647329 $\frac{a^2}{b^2} + \frac{b^2}{a^2} \geq \frac{a}{b}...

Đã gửi bởi Ngockhanh99k48 on 31-07-2016 - 13:27 trong Bất đẳng thức và cực trị

$\Leftrightarrow$ $a^4+b^4 \geq ab(a^2+b^2)$ $\Leftrightarrow$ $(a-b)^2[(a+b)^2-ab] \geq 0$



#647955 Tìm GTNN của $P=(a+b)(a+c)$

Đã gửi bởi Ngockhanh99k48 on 04-08-2016 - 18:18 trong Bất đẳng thức và cực trị

Theo giả thiết thì $abc(a+b+c)=1$.
Sử dụng AM-GM: $P=(a+b)(a+c)=a(a+b+c)+bc \geq 2\sqrt{abc(a+b+c)}=2$



#645014 Tìm max,min : P=$\frac{6-2(x-1)(y-1)}{(x-1)^{2...

Đã gửi bởi Ngockhanh99k48 on 15-07-2016 - 01:10 trong Hàm số - Đạo hàm

Đặt $a=x-1, b=y-1$, theo giả thiết thì $a^2+b^2=\frac{1}{2}$. Và P=12-4ab. Do $(a\sqrt{2})^2+(b\sqrt{2})^2=1$ nên tồn tại $\alpha \in [0; 2\pi]$ sao cho $a=\frac{\cos\alpha}{\sqrt{2}}, b=\frac{\sin\alpha}{\sqrt{2}}$. Khi đó: $P=12-2\sin\alpha.\cos\alpha=12-\sin 2\alpha$. Đến đây chắc bạn làm nốt được rồi đúng không? :)



#645015 CM: AM $\perp$ EF.

Đã gửi bởi Ngockhanh99k48 on 15-07-2016 - 01:21 trong Hình học

Trên tia đối của tia $BC$ lấy $G$ sao cho: $DE=BG$. Do đó $\triangle ADE =\triangle ABG$(c-g-c). Suy ra $AE=AG$ và $AE \perp AG$. Do đó $AF$ là phân giác góc $\widehat{EAF}$. Từ đó suy ra $\triangle AEF =\triangle AGF$. Khi đó $FA$ là phân giác góc $\widehat{EFG}$. $AM$ cắt $EF$ tại $K$ thì $\triangle AFK =\triangle AFB$(g-c-g). Từ đó $AM \perp EF$ tại $K$.



#645613 Chứng minh: $TP\parallel HQ.$

Đã gửi bởi Ngockhanh99k48 on 20-07-2016 - 09:16 trong Hình học

$BO$ cắt $(O)$ tại điểm thứ hai $S$. Khi đó ta cũng có $AHCS$ là hình bình hành (có các cạnh đối song song). Suy ra $H, Q, S$ thẳng hàng. Ta có $\triangle BDT \sim \triangle BAP (gg)$ và $\triangle BHD \sim \triangle BSA (gg)$. Từ đó ta có hệ thức $\frac{BT}{BP}=\frac{BD}{BA}=\frac{BH}{BS}$. Do đó $TP \parallel HQ$.



#647939 Chứng minh rằng: $2(x+y+z)-xyz\le 10$.

Đã gửi bởi Ngockhanh99k48 on 04-08-2016 - 16:56 trong Bất đẳng thức và cực trị

Bài 1: Theo mình không cần $xyz \leq 0$ đâu. Không mất tổng quát, giả sử $z= \max \{x, y, z \}$. Khi đó $18 = 2x^2+2y^2+2z^2 \geq 3(x^2+y^2) \geq 6xy$, suy ra $xy \leq 3$.
Áp dụng BĐT Cauchy-Schwarz ta có $[2(x+y+z)-xyz ] ^2 = [2(x+y)+z(2-xy)]^2$ $\leq [2^2+(2-xy)^2 ][(x+y)^2+z^2 ] = (9+2xy)(x^2y^2-4xy+8) = 100 + (2xy-7)(xy+2)^2 \leq 100$. Ta có đpcm. Dấu bằng xảy ra khi $x=z=2, y=-1$ và hoán vị.
P/s: bạn có thể xem thêm VNTST 2002



#647932 Chứng minh rằng: $2(x+y+z)-xyz\le 10$.

Đã gửi bởi Ngockhanh99k48 on 04-08-2016 - 16:30 trong Bất đẳng thức và cực trị

Lời giải bài 2: Đặt $x=3a, y=5b, z=2c$ với $a, b, c >0$. Theo giả thiết thì ta có: $3a+5b+7c=15abc$.
Ta có $P=6a+5b+4c$
Sử dụng AM-GM ta có: $(6a+5b+4c)^2(3a+5b+7c) \geq 225\sqrt[15]{a^{12}b^{10}c^8}. 15\sqrt[15]{a^3b^5c^7}=15^3abc$. Suy ra $P \geq 15$. Dấu bằng xảy ra khi: $x=3, y=4, z=2$



#658766 Chứng minh rằng: $c^2<2a^2+b^2$

Đã gửi bởi Ngockhanh99k48 on 22-10-2016 - 11:47 trong Hình học

$\widehat{C} = 2\widehat{A}+\widehat{B}$ nên $2\widehat{C}=180^{\circ}+\widehat{A}$ hay góc $C$ tù. Do đó $\cos C <0$. Bạn chứng minh đoạn cuối chưa chắc vô lí.



#662582 Tuần 4 tháng 11/2016 : Trục đẳng phương đi qua trực tâm

Đã gửi bởi Ngockhanh99k48 on 20-11-2016 - 23:39 trong Chuyên mục Mỗi tuần một bài toán Hình học

Lời giải :
Bài toán trên là sự kết hợp của hai bài toán con như sau:
Bài 1: Cho $\triangle LBC$ với trọng tâm $A$. Gọi $H$ là trực tâm $\triangle ABC$ và $HB, HC$ thứ tự cắt $AC, AB$ tại $E, F$. Khi đó $EF$ là trục đẳng phương của $(ABC)$ và $(LH)$.
Lời giải: Kẻ $LK \perp HB$, tức là $LK \perp CA$. Do $CA$ chia đôi $LB$ nên $F$ là trung điểm $KB$. Do đó $\mathbb{P}_{F/(ABC)}= \overline{FA}.\overline{FC}=-\overline{FB}.\overline{FH}=\overline{FK}.\overline{FH}=\mathbb{P}_{F/(LH)}$. Do đó $F$ thuộc trục đẳng phương của $(ABC)$ và $(LK)$. Tương tự xét điểm $E$ ta có đpcm.
Bài 2: Cho $\triangle ABC$ với trực tâm $H$ và nội tiếp $(O)$. $BH, CH$ cắt $AC, AB$ tại $E, F$. Gọi $d$ là đường thẳng qua $H$ song song $BC$. $EF$ cắt $d$ tại $L$. Gọi $K$ là hình chiếu của $L$ trên $OH$. Khi đó $H$ thuộc trục đẳng phương của đường tròn $(OK)$ và đường tròn Euler của $\triangle ABC$.
Lời giải: Gọi $M$ là trung điểm $BC$. $OM$ cắt $d$ tại $N$. Do đó $\overline{HO}.\overline{HK}=\overline{HL}.\overline{HN}$(1). Lấy $P$ thuộc $(O)$ sao cho $AP \parallel BC$. Kẻ $PQ \perp BC$ với $Q \in d$. Khi đó $BHQC$ là hình thang cân nên $Q \in (BHC)$. Xét phép nghịch đảo cực $H$, phương tích $\overline{HE}.\overline{HB}$: $\mathbb{I}_{H}^{\overline{HE}.\overline{HB}}$ biến $B \leftrightarrow E$, $C \leftrightarrow F$, $d \leftrightarrow d$. Do đó $L=EF \cap d \leftrightarrow (BHC) \cap d$. Suy ra $L, Q, E, B$ đồng viên. Suy ra $\overline{HN}.\overline{HL}=\frac{1}{2} \overline{HL}.\overline{HQ}=\frac{1}{2}.\overline{HE}.\overline{HB} = \mathbb{P}_{H/(Euler)}$(2). Từ (1) và (2) ta có đpcm.



#646492 Giải pt $2x^{2}+5x-1=7\sqrt{x^{3}-1}...

Đã gửi bởi Ngockhanh99k48 on 25-07-2016 - 21:58 trong Bất đẳng thức và cực trị

Viết lại như sau:
$2(x^2+x+1)+3(x-1)=7\sqrt{(x-1)(x^2+x+1)}$



#641859 CMR: Đường thẳng nối các trọng tâm vuông góc với đường thẳng nối các trực tâm.

Đã gửi bởi Ngockhanh99k48 on 23-06-2016 - 10:24 trong Hình học phẳng

Một tính chất cơ bản của tứ giác toàn phần, đường thẳng Gauss vuông góc với đường thẳng Steiner (đã được đề cập trong tài liệu chuyên Toán Hình học 10). Ta lại có, đường thẳng nối trọng tâm hai tam giác song song với đường thẳng Gauss của tứ giác 




#672134 Tuần 3 tháng 2/2017: Chứng minh tứ giác $AKNL$ ngoại tiếp

Đã gửi bởi Ngockhanh99k48 on 19-02-2017 - 20:55 trong Chuyên mục Mỗi tuần một bài toán Hình học

Lời giải: Dễ thấy rằng $DA$ và $DM$ đẳng giác trong góc $\angle EDF$. Ta lại có $MD. MN= ME. MF= MA. MJ$ nên $A, D, J, N$ đồng viên. Do $JN=JD$ nên $AN, AD$ đẳng giác trong góc $\angle BAC$. Kẻ đường cao $AH$. $AD$ cắt $(J)$ tại điểm thứ hai $X$. Khi đó $NX \parallel EF$. Ta có $\angle OAN = \angle HAD = 90^{\circ} - \angle CDX = 90^{\circ} - \angle NMF = \angle ANJ$.
Do đó tồn tại đường tròn $(O')$ tiếp túc trong cả hai đường tròn $(O)$ và $(J)$.
$AJ$ cắt $ON$ tại $I$. $O'I$ cắt $AN$ tại $P$. Gọi $\alpha_1, \alpha_2$ là đường tròn tâm $I$ tiếp xúc $AB, AC$ và $NL, NK$. Theo định lý $Monge-D'Alambert$, ta có $A$ là tâm vị tự ngoài của $\alpha_1$ và $(J)$, $N$ là tâm vị tự ngoài của $(J)$ và $(O')$, từ đó suy ra $P$ là tâm vị tự ngoài của $\alpha_1$ và $(O')$. Tương tự $P$ là tâm vị tự ngoài của $\alpha_2$ và $(O')$. Suy ra $\alpha_1 \equiv \alpha_2$. Như vậy $AKNL$ ngoại tiếp.



#670529 Tuần 1 tháng 2/2017: $QR$ đi qua điểm cố định khi $P$ di...

Đã gửi bởi Ngockhanh99k48 on 06-02-2017 - 00:06 trong Chuyên mục Mỗi tuần một bài toán Hình học

Cuối cùng diễn đàn đã mở lại :D

Lời giải:

Tiếp tuyến tại $A$ của $(O)$ cắt $BC$ tại $S$. Dễ thấy rằng $BE$ cắt $CF$ tại $K$ là đối xứng của $P$ qua $(O)$. $BR, CR$ thứ tự cắt $(O)$ tại $X$, $Y$. Theo giả thiết ta có $PX=PY$. Theo IMO Problem 4 ta có $SR=SA$. Mặt khác $AK \perp AR$, $AS \perp AO$ nên $\triangle AOK \stackrel{+}{\sim} \triangle ASR$. Kẻ tiếp tuyến $SZ$ khác $SA$ thì $Z$ cố định và $\angle AZR = \frac{1}{2} \angle ASR = \frac{1}{2} \angle AOK  = \angle APK = \angle AZK$ nên $KR$ đi qua $Z$. 

Ta có $ABZC$ là tứ giác điều hòa nên $K(AREF)=K(AZBC)=-1$, mặt khác theo tính chất hàng điểm cơ bản ta có $A(KQEF) = A(KQCB)=-1$ nên $A(KQEF)=K(AREF)$ hay $Q$ thuộc $KR$ do $E, F, Q$ thẳng hàng. Vậy $QR$ đi qua $Z$ cố định




#647857 ​$\frac{a}{a^2+2}+\frac{b}{...

Đã gửi bởi Ngockhanh99k48 on 03-08-2016 - 23:52 trong Bất đẳng thức và cực trị

Xử lí nhẹ :) $a^2+2 \geq 2a+1$. Do đó ta cần chứng minh $\frac{a}{2a+1}+\frac{b}{2b+1}+\frac{c}{2c+1} \leq 1$ $\Leftrightarrow$ $\frac{1}{2a+1}+\frac{1}{2b+1}+\frac{1}{2c+1} \geq 1$.
Giả thiết $abc=1$ nên tồn tại $x, y, z >0$ thỏa $a=\frac{x}{y}, b=\frac{y}{z}, c=\frac{z}{x}$. Từ đó, sử dụng Cauchy-schwarz ta có $\frac{1}{2a+1}+\frac{1}{2b+1}+\frac{1}{2c+1}=\frac{y}{2x+y}+\frac{z}{2y+z}+\frac{x}{2z+x} \geq \frac{(x+y+z)^2}{y(2x+y)+z(2y+z)+x(2z+x)}=1$.



#645356 CMR: $(a+b)^5 \geg 32ab$

Đã gửi bởi Ngockhanh99k48 on 17-07-2016 - 23:42 trong Bất đẳng thức và cực trị

Đề bài sai nhé, thử với $a=0,1$ và $b=0,2$.



#648201 Tìm GTNN của biểu thức: $P= \frac{a+3c}{a+b}+\frac{c+3b}{b+c}+...

Đã gửi bởi Ngockhanh99k48 on 06-08-2016 - 11:48 trong Bất đẳng thức và cực trị

Đề đúng hình như là: $P= \frac{a+3c}{a+b}+\frac{c+3a}{b+c}+\frac{4b}{c+a}$.

Đặt $a+b=z, b+c=x, c+a=y$ với $x, y,z >0$. Khi đó $a=\frac{y+z-x}{2}, b=\frac{x+z-y}{2}, c=\frac{x+y-z}{2}$.
Thay vào biểu thức ta có:
$P=\frac{x+2y-z}{z}+\frac{z+2y-x}{x}+\frac{2x+2z-2y}{y} =(\frac{x}{z}+\frac{z}{x})+2y(\frac{1}{z}+\frac{1}{x})+ \frac{2(x+z)}{y} -4$.
Sử dụng AM-GM ta có:
$P \geq 2 + \frac{8y}{x+z}+ \frac{2(x+z)}{y} -4 \geq 2+8-4 =6$. Dấu bằng xảy ra khi $x=y=z$ $\Leftrightarrow$ $a=b=c$.



#646495 Chứng minh $QB=PC$

Đã gửi bởi Ngockhanh99k48 on 25-07-2016 - 22:12 trong Hình học

Thay như bạn Le Nguyen Tan Kiet là đúng.
Lời giải: kẻ tiếp tuyến $SH$ của $(O)$. Khi đó tứ giác DBHC là tứ giác điều hòa nên $A(BCMH)=A(BCDH)=-1$. Do $M$ là trung điểm $BC$ nên $AH \parallel BC$. Cũng do tứ giác $DFHE$ điều hòa nên $A(MHQP)=A(DHFE)=-1$. Do $AH \parallel PQ$ nên $M$ là trung điểm $PQ$.



#646799 chứng minh: $\frac{(x+y)(x+z)}{4x}+\frac...

Đã gửi bởi Ngockhanh99k48 on 27-07-2016 - 20:55 trong Bất đẳng thức và cực trị

Cho các số dương $x,y,z$. Chứng minh :

$$\frac{(x+y)(x+z)}{4x}+\frac{(y+x)(y+z)}{4y}+\frac{(x+z)(z+y)}{4z}\geq x+y+z$$

$\Leftrightarrow$ $\frac{x(x+y+z)+yz}{x}+\frac{y(x+y+z)+xz}{y}+\frac{z(x+y+z)+xy}{z} \geq 4(x+y+z)$ $\Leftrightarrow$ $\frac{yz}{x}+\frac{xz}{y}+\frac{xy}{z} \geq x+y+z$. Sử dụng AM-GM ta có: $\frac{xy}{z}+\frac{yz}{x} \geq 2y$. Tương tự ta có đpcm. 




#660101 Tuần 1 tháng 11/2016 : Trục đẳng phương đi qua giao điểm

Đã gửi bởi Ngockhanh99k48 on 31-10-2016 - 20:45 trong Chuyên mục Mỗi tuần một bài toán Hình học

Một hướng đi gần giống:
$AK$ cắt $(O)$ tại $E$. $PK$ cắt $(BOC)$ tại $F$. Ta có $\overline{KE}.\overline{KA}=\overline{KB}.\overline{KC}=\overline{KP}.\overline{KF}$ nên $A, P, E, F$ đồng viên. Do đó $\widehat{PAK}=\widehat{PFA}$ (vì $PA=PE$). Suy ra $PA$ là tiếp tuyến của $(AKF)$.
$AP$ cắt $(BOC)$ tại $Q$. $QF, PE$ cắt $BC$ tại $M, N$. Do $AK \parallel PT$ nên $PT$ là phân giác của góc $\widehat{EPQ}$. Nếu $PE$ cắt $(BOC)$ tại $S$ thì $SQ \parallel BC$. Bằng cộng các cung chứa góc của đường tròn $(BOC)$ suy ra $\widehat{PFQ}=\widehat{ENM}$ hay $M, F, N, P$ đồng viên. Suy ra $\widehat{QMN}=\widehat{FPE}=\widehat{FAK}$ do đó $A, K, F, M$ đồng viên.
$(AKM)$ cắt $AD$ tại $Z$ thì ta có $\widehat{KLD}=\widehat{KAP}=\widehat{AMK}=\widehat{KZD}$ nên $Z \in (KLD)$. Do đó $\widehat{ZFP}=\widehat{DAK}=\widehat{ZDP}$ hay $Z, F, P, D$ đồng viên. Đến đây ta có đpcm.